PT4.S4.Q18 - Zelda: Dr. Ladlow, a research psychologist, has

A Girl Knows NothingA Girl Knows Nothing Alum Member
edited June 2019 in Logical Reasoning 68 karma

help

So I was stuck between A and B. Had I not seen this was a 3/5 difficulty I would have chosen A without hesitation. I still choose A, but contemplated B quite a bit.

Can someone please explain how A is wrong and B is correct?

Thank you.

Comments

  • BlindReviewerBlindReviewer Alum Member
    855 karma

    There is no attack on character here -- you may be able to think that from the fact that Anson says Ludlow "is not a responsible psychologist" but that's not just a random attack on Ludlow's character. He supports it with the general principle that "If you were a responsible psychologist, you would be open to the idea that new evidence might prove your theory wrong." It's general because it is applied to "responsible psychologists" broadly.

  • A Girl Knows NothingA Girl Knows Nothing Alum Member
    68 karma

    Thank you so much!

  • MistaTee001MistaTee001 Member
    105 karma

    Hello,

    This question took me a while to understand why A was not the answer. Anson concludes that Dr. Ladlow isn't a responsible psychologist. The question stem asks: "Anson bases his conclusion about Dr. Ladlow based on which of the following?"

    A ) If anything, the attack on his character would be the conclusion, not the support for the conclusion. Furthermore, from what I understand a personal attack would be more along the lines of: "Dr. Ladlow smokes cigarettes; we shouldn't believe anything he says"; not a professional criticism.

    B ) Is correct because it takes the general principle within the stimulus of that responsible psychologists need to consider the potential of evidence that could refute their own findings, which Dr. Ladlow fails to do. Thus by failing to adhere to a general principle, Anson states that Dr. Ladlow's incorrect.

    C ) There's no ambiguous term within this stimulus.

    D ) Anson doesn't dispute Dr. Ladlow's facts (i.e. that the Dr.'s theory about rats isn't correct, its just that Anson adds to the notion that he must also consider the possibly that it might NOT be correct)

    E ) Anson doesn't reject the Dr.'s theoretical explanation.

Sign In or Register to comment.